No hermiticidad de Dirac Lagrangiano: ¿ímpetu nulo?

El dirac lagrangiano habitual es L ( ψ , ψ ¯ ) = ψ ¯ ( i ∂̸ metro ) ψ . Los momentos canónicos son

π = L ψ , 0 = i ψ π ¯ = L ψ ¯ , 0 = 0

El hecho de que π ¯ = 0 no se discute en ninguna parte en los libros que he leído. Si ignoramos esto y seguimos adelante, podemos escribir { ψ ( t , X ) , π ( t , y ) } = i d ( X y ) , que de hecho es equivalente a { ψ ( t , X ) , ψ ( t , y ) } = d ( X y ) .

Podemos escribir la solución de la MOE como

ψ ( X ) = s d 3 pag ( 2 π ) 3 2 mi   b pag s tu s ( pag ) mi i pag X + C pag s v s ( pag ) mi i pag X
dónde b , C son operadores de creación-aniquilación.

Además, probamos que, por ejemplo, { b s ( pag ) , b t ( q ) } = ( 2 π ) 3 d s t d ( pag q ) y una relación similar para C pag . Después de eso, demostramos fácilmente que { H , C pag } = ω pag C pag , Lo que significa que | pag = C pag | 0 , y H | pag = ω pag | pag , es decir, | pag tiene energía ω pag .

Hasta ahora, todo bien. Sin embargo, si tuviéramos que estudiar esta teoría desde los primeros principios, también deberíamos imponer { ψ ¯ ( t , X ) , π ¯ ( t , y ) } = i d ( X y ) , lo cual es imposible como π ¯ = 0 (Deberíamos imponer este conmutador porque se supone que los campos están ψ y ψ ¯ : deben considerarse como variables independientes)

Es fácil ver que esto está relacionado con el hecho de que el lagrangiano no es hermitiano. Podemos arreglar esto restando la derivada total i 2 m ( ψ ¯ γ m ψ ) ; terminamos con

L ( ψ , ψ ¯ ) = i 2 ψ ¯ ∂̸ ψ i 2 m ψ ¯ γ m ψ ψ ¯ metro ψ

De este nuevo lagrangiano obtenemos la misma PDE (ecuación de Dirac), como deberíamos esperar. Además, los momentos conjugados son ambos distintos de cero:

π = L ψ , 0 = i 2 ψ π ¯ = L ψ ¯ , 0 = i 2 ψ

No es difícil ver que esto da lugar al mismo hamiltoniano que antes. El problema viene del 1 2 tener en cuenta π , π ¯ : aparecen aquí y allá. Por ejemplo, las relaciones canónicas de conmutación se cambian a { ψ ( t , X ) , ψ ( t , y ) } = 2 d ( X y ) , que, a su vez, son equivalentes a { b s ( pag ) , b t ( q ) } = 2 ( 2 π ) 3 d s t d ( pag q ) y { H , C pag } = 2 ω pag C pag .

Esta última relación es horrible: un estado | pag tiene energía igual a 2 ω pag . Siento que ambos enfoques son insatisfactorios, al menos hasta cierto punto. El primero es el enfoque habitual, pero el lagrangiano no es hermitiano y el impulso se conjuga con ψ ¯ es nulo, por lo que no es posible un uso sistemático del CCR. El segundo soluciona ambos problemas, tiene la misma ecuación de movimiento y el mismo hamiltoniano, pero los CCR son incorrectos por un factor de dos, lo que hace que la energía de las partículas sea 2 ω pag en lugar de ω pag .

Sé que hay muchas preguntas en SE similares a esta, como

¿Relación de anticonmutación de signo incorrecto para el campo de Dirac?

Ecuación de Dirac como sistema hamiltoniano

etc.

pero estas personas estaban preguntando acerca de señales u otros temas. Mi pregunta es sobre los factores de 2 que aparece de vez en cuando, cambiando el espectro de energía de la teoría.

Finalmente, está esta pregunta From Lagrangian to Hamiltonian in Fermionic Model que se acerca bastante, pero la pregunta no está clara (algunos índices libres en el lagrangiano) y OP dice que el hamiltoniano cambia con el nuevo lagrangiano. Fácilmente se puede ver que esto no es cierto: el hamiltoniano es el mismo, pero los CCR son diferentes (y esto no se aborda). Además, las respuestas afirman que los momentos no cambian, lo que creo que no es cierto. También dicen que los campos son tales que ψ / ψ ¯ , 0 0 , y esto no se explica más (y ni siquiera sé si es cierto o no).

Entonces, ¿alguien puede arrojar algo de luz sobre esto? ¿Por qué aceptamos el primer enfoque independientemente de sus fallas? ¿No debería el hecho de que π ¯ = 0 despertar alguna sospecha? ¿Por qué el segundo enfoque no funciona correctamente?

Respuestas (1)

OP pregunta cómo realizar la transformación singular de Legendre para una teoría lagrangiana de los fermiones de Dirac. Esto ya se hizo en mi respuesta Phys.SE aquí usando el método Faddeev-Jackiw . Sin embargo, OP quiere considerar el tradicional Dirac-Bergmann 1 análisis. Las posibles complicaciones se enumeraron en mi respuesta Phys.SE aquí .

Partimos de lo manifiestamente real 2 Densidad lagrangiana

(1) L   =   i 2 m = 0 3 ( ψ ¯ γ m m ψ m ψ ¯   γ m ψ ) metro ψ ¯ ψ   =   L 0 H ,

dónde

(2) L 0   :=   i 2 ( ψ ψ ˙ ψ ˙ ψ ) , H   :=   i 2 j = 1 3 ( ψ ¯ γ j j ψ j ψ ¯   γ j ψ ) + metro ψ ¯ ψ .

El primer problema es que los campos complejos

(3) ψ α     ( ψ α 1 + i ψ α 2 ) / 2 y ψ α     ( ψ α 1 i ψ α 2 ) / 2 ,

no son campos independientes. Probablemente sea más convincente si reescribimos la teoría (1) usando las partes real e imaginaria, ψ 1 y ψ 2 , que son dos campos independientes de valores reales. Entonces

(4) L 0   =   i 2 a = 1 2 ( ψ a ) T ψ ˙ a .

Ahora defina momentos canónicos de valor imaginario

(5) π a α   :=   R L ψ ˙ α a   =   i 2 ψ α a

como el derecho 3 derivado de L wrt. ψ ˙ α a . Los corchetes de super-Poisson canónicos de tiempo igual distintos de cero dicen

(6) { ψ α a ( X , t ) , π b β ( y , t ) } PAG B   =   d b a d α β   d 3 ( X y ) .

[Curiosamente, el corchete canónico de Poisson { , } en sí misma es intrínsecamente imaginaria en el sector fermiónico.] Eq. (5) produce dos restricciones principales

(7) x a   :=   π a i 2 ψ a     0 ,

que son restricciones de segunda clase

(8) Δ a b α β ( X y )   :=   { x a α ( X , t ) , x b β ( y , t ) } PAG B   =   i d a b   d α β   d 3 ( X y ) ,

con matriz inversa

(9) ( Δ 1 ) α β a b ( X y )   =   i d a b   d α β   d 3 ( X y ) .

El soporte Poisson (6) debe ser reemplazado por el soporte Dirac . Los corchetes fundamentales de Dirac decían

(10) { ψ α a ( X , t ) , ψ β b ( y , t ) } D B   =   i d a b   d α β   d 3 ( X y ) .

El resultado (10) está de acuerdo con el método de Faddeev-Jackiw, véase, por ejemplo, eq. (5 ') en mi respuesta Phys.SE aquí , que también enumera las relaciones canónicas anticonmutación (CAR) correspondientes.

Referencias:

  1. A. Das, Conferencias sobre QFT, (2008); Capítulo 10.

--

1 El análisis de Dirac-Bergmann de los campos impares de Grassmann también se considera en mi respuesta Phys.SE aquí .

2 Se puede demostrar que la densidad lagrangiana (1) es real usando

(11) ( γ m )   =   γ 0 γ m γ 0 , ( γ 0 ) 2   =   1 .

Convenciones: En esta respuesta, usaremos ( + , , , ) Convención de signos de Minkowski y álgebra de Clifford

(12) { γ m , γ v } +   =   2 η m v 1 4 × 4 .

3 Las derivadas izquierda y derecha de las variables impares de Grassmann también se analizan en mi respuesta Phys.SE aquí .